If f is meromorphic on U with only a finite number of poles, then

  • Thread starter Thread starter fauboca
  • Start date Start date
  • Tags Tags
    Finite Poles
Click For Summary
If a function f is meromorphic on a domain U with only a finite number of poles, it can be expressed as the quotient of two analytic functions, g and h. The poles are defined as points where f approaches infinity, and around each pole, a specific behavior is observed, allowing for the function to be represented as f = g/(z - z_0) for a single pole. The function h must be analytic in U and have zeros at the poles of f, ensuring that the product fh has only removable singularities. This relationship confirms that f can be represented as f = g/h, where h is designed to eliminate the singularities caused by the poles. The discussion emphasizes the importance of constructing h correctly to maintain analyticity in U.
fauboca
Messages
157
Reaction score
0
If f is meromorphic on U with only a finite number of poles, then f=\frac{g}{h} where g and h are analytic on U.

We say f is meromorphic, then f is defined on U except at discrete set of points S which are poles. If z_0 is such a point, then there exist m in integers such that (z-z_0)^mf(z) is holomorphic in a neighborhood of z_0.

A pole is \lim_{z\to a}|f(z)| =\infty.

S0 the trouble is showing that f is the quotient of two holomorphic functions.
 
Physics news on Phys.org


How would you show it if there is only one pole?
 


Dick said:
How would you show it if there is only one pole?

If the pole is at z_0, then

f=\frac{g}{(z-z_0)}

Let S be the set of the poles of f in U and let h be analytic in U with singularities at the points in S. Let the order of the singularity be the order of the pole in f. Then fh has only removable singularities in U.

fh=g\Rightarrow f=\frac{g}{h}
 
Last edited:


fauboca said:
If the pole is at z_0, then

f=\frac{g}{(z-z_0)}

Let S be the set of the poles of f in U and let h be analytic in U with singularities at the points in S. Let the order of the singularity be the order of the pole in f. Then fh has only removable singularities in U.

fh=g\Rightarrow f=\frac{g}{h}

You want h to have 'zeroes', not 'singularities'.
 
Question: A clock's minute hand has length 4 and its hour hand has length 3. What is the distance between the tips at the moment when it is increasing most rapidly?(Putnam Exam Question) Answer: Making assumption that both the hands moves at constant angular velocities, the answer is ## \sqrt{7} .## But don't you think this assumption is somewhat doubtful and wrong?

Similar threads

  • · Replies 2 ·
Replies
2
Views
2K
  • · Replies 1 ·
Replies
1
Views
1K
  • · Replies 6 ·
Replies
6
Views
3K
  • · Replies 2 ·
Replies
2
Views
2K
  • · Replies 5 ·
Replies
5
Views
2K
  • · Replies 10 ·
Replies
10
Views
2K
  • · Replies 21 ·
Replies
21
Views
3K
  • · Replies 2 ·
Replies
2
Views
2K
  • · Replies 2 ·
Replies
2
Views
1K
  • · Replies 1 ·
Replies
1
Views
2K